LSAT and Law School Admissions Forum

Get expert LSAT preparation and law school admissions advice from PowerScore Test Preparation.

User avatar
 Dave Killoran
PowerScore Staff
  • PowerScore Staff
  • Posts: 5852
  • Joined: Mar 25, 2011
|
#88298
Complete Question Explanation
(The complete setup for this game can be found here: lsat/viewtopic.php?f=177&p=88290#p88290)

The correct answer choice is (B).

The condition in the question stem assigns O to the second-place team. As O is one of the three candidates for the third-place team, removing O from consideration forces M and N on the third-place team. Thus, P and S must be on the first place team.

Because S is on the first-place team, from the first rule G must then be the first-place team, leading to the following setup:

G4-Q20-d1.png

Accordingly, answer choice (B) is correct.
You do not have the required permissions to view the files attached to this post.

Get the most out of your LSAT Prep Plus subscription.

Analyze and track your performance with our Testing and Analytics Package.